Proving Constant Sequence of Positive Integers Starting with n

Click For Summary
SUMMARY

The discussion centers on proving that a sequence defined by a positive integer n, where a_1 = n and subsequent terms a_k are determined by the divisibility of the sum of the sequence by k, eventually becomes constant. The participants analyze the proof structure, identifying flaws in the assumption that n can vary and emphasizing the need for a fixed n. They conclude that for sufficiently large k, the sum of the sequence will converge, thus validating the constancy of the sequence.

PREREQUISITES
  • Understanding of sequences and series in mathematics
  • Familiarity with divisibility rules and modular arithmetic
  • Knowledge of proof techniques, including proof by contradiction
  • Basic concepts of least common multiples (LCM)
NEXT STEPS
  • Study the properties of sequences and their convergence
  • Learn about modular arithmetic and its applications in number theory
  • Explore proof techniques in mathematics, focusing on contradictions and direct proofs
  • Investigate the significance of least common multiples in number theory
USEFUL FOR

Mathematics students, particularly those preparing for competitions like the USA Mathematics Olympiad, and educators looking to enhance their understanding of sequence behavior and proof strategies.

Feldoh
Messages
1,336
Reaction score
3
Let n be a positive integer. Define a sequence by setting a_1 = n and, for each k > 1, letting a_k be the unique integer in the range 0 \le a_k \le k-1[/itex] for which \displaystyle a_1 + a_2 + \cdots + a_k is divisible by k. For instance, when n = 9 the obtained sequence is \displaystyle 9, 1, 2, 0, 3, 3, 3, \ldots. Prove that for any n the sequence \displaystyle a_1, a_2, a_3, \ldots eventually becomes constant.
 
Physics news on Phys.org
that's cool, where'd you find this?
 
Thanks, that was fun! I too would like to know where you found this problem!

Here is my attempt at a solution:

It is clear that once {\sum}^k_{i=1}{a_i}=ck for some c<k, subsequent terms in the sequence simply require additional c's to be added to them. Thus, we need to prove that for any starting n, there exists a k such that {\sum}^k_{i=1}{a_i}=ck for some c<k.

Assume not. That is, \forall{n} and \forall{k} (where n and k are positive integers), {\sum}^k_{i=1}{a_i}=Ck for some C\geq{k}.

Then we have

{k}^{2}\leq{Ck}=n+{a_2}+{a_3}+...+{a_k}\leq{n+1+2+...+(k-1)}={n+\frac{k(k+1)}{2}}=\frac{2n+k^{2}-k}{2}}

So by letting {n}\geq{\frac{k^{2}+k}{2}} for an arbitrary k, we derive a contradiction, for the sequence starting with n converges by the kth term. This completes the proof.



Is this correct?
 
There is one flaw in the proof, mainly I think to do with how you've written it: n is fixed, you can't choose n; it doesn't make sense to say "let n be greater than (k^2+k)/2 for an arbitrary k".

It is also an unnecessary proof by contradiction. It suffices to note that given any n then for k sufficiently large n + k(k-1)/2 is less than k^2.
 
Aaah, I see what you mean. Thanks for the correction!
 
haha, that's a usa math olympiad question. ah, reminds me of the good old days. looking forward to putnam in college
 
Feldoh said:
Let n be a positive integer. Define a sequence by setting a_1 = n and, for each k &gt; 1, letting a_k be the unique integer in the range 0 \le a_k \le k-1[/itex] for which \displaystyle a_1 + a_2 + \cdots + a_k is divisible by k. For instance, when n = 9 the obtained sequence is \displaystyle 9, 1, 2, 0, 3, 3, 3, \ldots. Prove that for any n the sequence \displaystyle a_1, a_2, a_3, \ldots eventually becomes constant.
<br /> define A(n) to be the least common multiple of the set {1,2,3...n} <br /> A(n) = {1,2,6,12,60,420,...}<br /> I noticed that if A_{m-1} &amp;lt; n \le A_{m} then {\sum}^k_{i=1}{a_i} \geq \frac{kA_{m}}{m} and a_{i} converges to \frac{A_{m}}{m} by the value of k equal to m. I thought I had a proof but need to come back to this later.
 
matticus said:
that's cool, where'd you find this?

It's actually from 2007 USA Mathematics Olympiad.

I found them here the solutions are there too. I had a lot of fun with the first one so I though I'd post it :biggrin:
http://www.artofproblemsolving.com/Wiki/index.php/2007_USAMO
 
Last edited by a moderator:
matt grime said:
There is one flaw in the proof, mainly I think to do with how you've written it: n is fixed, you can't choose n; it doesn't make sense to say "let n be greater than (k^2+k)/2 for an arbitrary k".

It is also an unnecessary proof by contradiction. It suffices to note that given any n then for k sufficiently large n + k(k-1)/2 is less than k^2.

As I was falling asleep just now, this problem popped into my head, and I realized that I made an extremely stupid error, and if I had not made that error, I would have never made the mistake you pointed out (and I would have, presumably, had a wholly correct proof).

The error? I stated the negation incorrectly...I said "for all n" when I should have said "there exists an n". What a silly mistake. Oh well, its summer, I need to get these bloopers out of my system before the semester begins :rolleyes:
 
  • #10
abelian jeff said:
The error? I stated the negation incorrectly...I said "for all n" when I should have said "there exists an n".

No you shouldn't. n is given to you in the question. k is what you play with.
 
  • #11
Yes, it turns out that all you need to work with is the k. However, I said this:

Assume not. That is, \forall{n} and \forall{k} (where n and k are positive integers), {\sum}^k_{i=1}{a_i}=Ck for some C\geq{k}.

That is clearly incorrect.
 

Similar threads

  • · Replies 10 ·
Replies
10
Views
3K
Replies
4
Views
1K
  • · Replies 4 ·
Replies
4
Views
2K
  • · Replies 1 ·
Replies
1
Views
2K
  • · Replies 25 ·
Replies
25
Views
3K
  • · Replies 3 ·
Replies
3
Views
2K
  • · Replies 10 ·
Replies
10
Views
3K
  • · Replies 2 ·
Replies
2
Views
2K
  • · Replies 15 ·
Replies
15
Views
5K
  • · Replies 1 ·
Replies
1
Views
1K